Только магнитное поле действует на: какой опыт доказывает ,что магнитное поле действует только на движущиеся заряды?

Содержание

Действие магнитного поля на проводник с током — урок. Физика, 8 класс.

Рассмотрим рис. \(1\). К источнику тока подсоединены два пластинчатых проводника.

а)

  • цепь не замкнута;
  • в проводниках тока нет;
  • проводники не взаимодействуют друг с другом;

б)

  • провода соединены последовательно;
  • ток по проводникам идёт в противоположных направлениях;
  • проводники отталкиваются друг от друга;

в)

  • проводники соединены параллельно;
  • направление силы тока в проводниках совпадает;
  • проводники притягиваются друг к другу.

  

Рис. 1. Изображение проводников, подсоединенных к источнику тока

 

Обрати внимание!

Магнитное поле действует с некоторой силой на любой проводник с током, находящийся в этом поле.

 

Подвесим металлический проводник на гибких проводах, присоединённых к источнику тока.

Для демонстрации воздействия магнитного поля на участок проводника с током соберём установку из подковообразного магнита — источника постоянного магнитного поля и проводника, подключённого к источнику тока (рис. \(2\)). С помощью реостата будем управлять величиной тока в цепи.

 

Рис. 2. Изображение отсутствия отклонения проводника вблизи магнита при разомкнутом ключе

Рис. 3. Изображение отклонения проводника вблизи магнита при увеличении силы тока в цепи 

  

1. Замкнём цепь. По участку провода, находящемуся в поле постоянного магнита, пройдёт ток, направление которого зависит от полюсов источника тока, к которым подключены концы провода. Вектор магнитной индукции \(\vec{B}\) направлен от северного полюса к южному — сверху вниз. Ток в проводнике направлен от наблюдателя. Магнитное поле втягивает проводник с током (рис. \(3\)).

 

2. Изменим направление тока, поменяв полюса источника тока. Тогда проводник будет выталкиваться магнитным полем.

 

3. Если полюса магнита поменять (перевернуть магнит), то направление движения проводника изменится на противоположное.

Правило левой руки

Ладонь левой руки нужно разместить так, чтобы линии магнитной индукции \(\vec{B}\) входили в ладонь, четыре вытянутых пальца показывали направление движения тока \(\vec{I}\) в проводнике, тогда отогнутый под прямым углом большой палец покажет направление действия силы Ампера \(\vec{F_A}\), действующей на проводник с током.

Движение проводника вызвано этой силой \(\vec{F_A}\), поэтому направление движения проводника совпадает с направлением действия силы \(\vec{F_A}\) (рис. \(4\)).

 

Рис. 4. Изображение положения руки в пространстве при определении направления силы Ампера

Источники:

Рис. 1. Указание авторства не требуется: © ЯКласс.

Рис. 2. Указание авторства не требуется: © ЯКласс.

Рис. 3. Указание авторства не требуется: © ЯКласс.

Рис. 4. Указание авторства не требуется: © ЯКласс.

Действие магнитного поля на ток. Правило левой руки.

Поместим между полюсами магнита проводник, по кото­рому протекает постоянный электрический ток. Мы тотчас же заметим, что проводник будет выталкиваться полем магнита из междуполюсного пространства.

Объяснить это можно следующим образом. Вокруг провод­ника с током (Рисунок 1.) образуется собственное магнитное поле, силовые линии которого по одну сторону проводника направ­лены так же, как и силовые линии магнита, а по другую сто­рону проводника — в противопо­ложную сторону. Вследствие это­го с одной стороны проводника (на рисунке 1 сверху) маг­нитное поле оказывается сгущен­ным, а с другой его стороны (на рисунке 1 снизу) – разрежен­ным. Поэтому проводник испыты­вает силу, давящую на него вниз. И если проводник не закреплен, то он будет перемещаться.

Рисунок 1. Действие магнитного поля на ток.

Правило левой руки

Для быстрого определения направления движения провод­ника с током в, магнитном поле существует так называемое правило левой руки (рисунок 2.).

Рисунок 2. Правило левой руки.

Правило левой руки состоит в следую­щем: если поместить левую руку между полюсами маг­нита так, чтобы магнитные силовые линии входили в ладонь, а четыре пальца ру­ки совпадали с направлением тока в проводнике, то боль­шой палец покажет направ­ление движения проводника.

Итак, на проводник, по которому протекает электри­ческий ток, действует сила, стремящаяся перемещать его перпендикулярно магнитным силовым линиям. Опытным путем можно определить величину этой силы. Оказы­вается, что сила, с которой магнитное поле действует на проводник с током, прямо пропорциональна силе тока в проводнике и длине той части проводника, которая нахо­дится в магнитном поле (рисунок 3 слева).

Это правило справедливо, если проводник расположен под прямым углом к магнитным силовым линиям.

Рисунок 3. Сила взаимодействия магнитного поля и тока.

Если же проводник расположен не под прямым углом к магнитным силовым линиям, а, например, так, как изобра­жено на рисунке 3 справо, то сила, действующая на проводник, будет пропорциональна силе тока в проводнике и длине проекции части проводника, находящейся в магнитном поле, на плос­кость, перпендикулярную магнитным силовым ли­ниям. Отсюда следует, что если проводник паралле­лен магнитным силовым линиям, то сила, дейст­вующая на него, равна нулю.

Если же проводник перпендикулярен направ­лению магнитных силовых линий, то сила, действую­щая на него, достигает наибольшей величины.

Сила, действующая на проводник с током, зави­сит еще и от магнитной индукции. Чем гуще рас­положены магнитные си­ловые линии, тем больше сила, действующая на проводник с током.

Подводя итог всему изложенному выше, мы можем действие магнитного поля на проводник с током выразить следующим правилом:

Сила, действующая на проводник с током, прямо пропорциональна магнитной индукции, силе тока в проводнике и длине проекции части проводника, находящейся в магнитном поле, на плоскость, перпендикулярную маг­нитному потоку.

Необходимо отметить, что действие магнитного поля на ток не зависит ни от вещества проводника, ни от его сечения. Дей­ствие магнитного поля на ток можно наблюдать даже при от­сутствии проводника, пропуская, например, между полюсами магнита поток быстро несущихся электронов.

Действие магнитного поля на ток широко используется в науке и технике.

На использовании этого действия основано устройство электродвигателей, превращающих электрическую энергию в механическую, устройство магнитоэлектрических приборов для измерения напряжения и силы тока, электроди­намических громкоговорителей, превращающих электрические колебания в звук, специальных радиоламп — магнетронов, катодно-лучевых трубок и т. д. Действием магнитного поля на ток пользуются для измерения массы и заряда электрона и даже для изучения строения вещества.

ПОНРАВИЛАСЬ СТАТЬЯ? ПОДЕЛИСЬ С ДРУЗЬЯМИ В СОЦИАЛЬНЫХ СЕТЯХ!

Похожие материалы:

Добавить комментарий

Магнитное поле помогает оздоровлению мышц. В спортзал можно не ходить?

Автор фото, Getty Images

Недавнее исследование, проведенное учеными из Национального университета Сингапура, показало, что один из белков, входящих в состав нашей мышечной ткани, реагирует на слабое магнитное поле, стимулируя мышечный рост.

С возрастом люди постепенно теряют мышечную массу и силу. Причины этого до сих пор толком не известны, поэтому изучение всех аспектов мышечного здоровья представляет немалый интерес, как для ученых, так и для всех, кто столкнулся с проблемой возрастной потери мышечной массы.

Команда под руководством доцента Альфредо Франко-Обрегона из Института инноваций и технологий здравоохранения при сингапурском университете (iHealthtech) обнаружила, что белок TRPC1 реагирует на слабые колебания магнитного поля.

Такая реакция обычно наблюдается во время физических упражнений. Эту чувствительность к воздействию магнитного поля можно использовать для стимуляции восстановления мышц, что может улучшить качество жизни пациентов с нарушенной подвижностью.

Результаты совместного исследования ученых из сингапурского университета и швейцарского Федерального технологического института опубликованы в журнале Advanced Biosystems.

Магнитное поле и здоровье мышц

Магнитные поля, которые исследователи использовали для стимуляции мышц, всего в 10-15 раз сильнее, чем магнитное поле Земли, но намного слабее, чем привычный нам стержневой магнит. Это дало ученым возможность предположить, что мышцы человека естественным образом реагируют на слабое магнитное поле.

Чтобы проверить эту теорию, группа исследователей сначала использовала специальную экспериментальную установку, чтобы нейтрализовать влияние всех окружающих магнитных полей. Исследователи обнаружили, что мышечные клетки действительно росли медленнее, когда они были защищены от воздействия всех магнитных полей окружающей среды.

Автор фото, Westend61

Подпись к фото,

Об отказе от физических упражнений в исследовании не говорится – какая незадача для лентяев…

Эти наблюдения убедительно подтвердили идею о том, что магнитное поле Земли естественным образом взаимодействует с мышцами, вызывая биологические реакции.

Чтобы продемонстрировать участие TRPC1 в качестве своего рода “антенны”, реагирующей на магнитное поле, исследователи с помощью генной инженерии создали мышечные клетки, из генома которых был удален белок TRPC1.

Оказалось, что клетки-мутанты не реагируют на любое магнитное поле. Затем исследователи смогли восстановить магнитную чувствительность путем избирательной доставки TRPC1 к этим клеткам.

Метаболические изменения, аналогичные тем, которые достигаются при физических упражнениях, наблюдались в предыдущих клинических испытаниях и исследованиях, проведенных доцентом Франко-Обрегоном. Как оказалось, для стимуляции мышечных клеток достаточно воздействия магнитного поля в течение всего 10 минут в неделю.

Почему это важно

Здоровье мышц сильно влияет на общее метаболическое состояние человека – вес, уровень сахара в крови, инсулина и холестерина, и в целом на самочувствие человека. Особенно это важно для людей с различными заболеваниями, которым трудно поддерживать высокий уровень физической активности в повседневной жизни.

Магнитные поля, симулируя работу мышц, могут помочь пациентам, неспособным выполнять упражнения из-за травм, болезней или слабости. Сейчас ученые исследуют возможности снижения зависимости пациентов от лекарственных препаратов при лечении таких заболеваний, как диабет.

“Мы надеемся, что наши исследования помогут снизить количество прописываемых препаратов для лечения заболеваний, и таким образом уменьшить побочные эффекты от лекарств и повысить качество жизни пациентов”, – говорит Франко-Обрегон.

Урок 3. магнитная индукция. действие магнитного поля на проводник с током и движущуюся заряженную частицу – Физика – 11 класс

Физика, 11 класс

Урок 3. Магнитная индукция. Действие магнитного поля на проводник и движущуюся заряжённую частицу

Перечень вопросов, рассматриваемых на уроке:

1) магнитное поле;

2) вектор магнитной индукции, линии магнитной индукции;

3) сила Ампера, сила Лоренца;

4) правило буравчика, правило левой руки.

Глоссарий по теме

Магнитная индукция – векторная величина, характеризующая величину и направление магнитного поля.

Сила Ампера – сила, действующая со стороны магнитного поля на проводник с током.

Сила Лоренца – сила, действующая со стороны магнитного поля на движущую частицу с зарядом.

Правило «буравчика» – правило для определения направления магнитного поля проводника с током.

Правило левой руки – правило для определения направления силы Ампера и силы Лоренца.

Соленоид – проволочная катушка.

Рамка с током – небольшой длины катушка с двумя выводами из скрученного гибкого проводника с током, способная поворачиваться вокруг оси, проходящей через диаметр катушки.

Основная и дополнительная литература по теме урока

Мякишев Г.Я., Буховцев Б.Б.,. Чаругин В.М. Физика.11 класс. Учебник для общеобразовательных организаций. М.: Просвещение, 2014. – С. 3 – 20

2. А.П. Рымкевич. Сборник задач по физике. 10-11 классы. – М: Дрофа, 2009. – С.109 – 112

Основное содержание урока

Магнитное поле – особый вид материи, которая создаётся электрическим током или постоянными магнитами. Для демонстрации действия и доказательства существования магнитного поля служат магнитная стрелка, способная вращаться на оси, или небольшая рамка (или катушка) с током, подвешенная на тонких скрученных гибких проводах.

Рамка с током и магнитная стрелка под действием магнитного поля поворачиваются так, что северный полюс (синяя часть) стрелки и положительная нормаль рамки указывают направление магнитного поля.

Магнитное поле, созданное постоянным магнитом или проводником с током, занимает всё пространство в окрестности этих тел. Магнитное поле принято (удобно) изображать в виде линий, которые называются линиями магнитного поля. Магнитные линии имеют вихревой характер, т.е. линии не имеют ни начала, ни конца, т.е. замкнуты. Направление касательной в каждой точке линии совпадает с направлением вектора магнитной индукции. Поля с замкнутыми линиями называются вихревыми.

Магнитное поле характеризуется векторной величиной, называемой магнитной индукцией. Магнитная индукция характеризует «силу» и направление магнитного поля – это количественная характеристика магнитного поля.

Она обозначается символом За направление вектора магнитной индукции принимают направление от южного полюса к северному магнитной стрелки, свободно установившейся в магнитном поле.

Направление магнитного поля устанавливают с помощью вектора магнитной индукции.

Направление вектора магнитной индукции прямого провода с током определяют по правилу буравчика (или правого винта).

Правило буравчика звучит следующим образом:

если направление поступательного движения буравчика совпадает с направлением тока в проводнике, то направление вращения ручки буравчика совпадает с направлением линий магнитного поля тока.

Направление магнитного поля внутри соленоида определяют по правилу правой руки.

Определим модуль вектора магнитной индукции.

Наблюдения показывают, что максимальное значение силы, действующей на проводник, прямо пропорционально силе тока, длине проводника, находящегося в магнитном поле.

F_max ~ I; F ~ Δl.

Тогда, зависимость силы от этих двух величин выглядит следующим образом

Отношение зависит только от магнитного поля и может быть принята за характеристику магнитного поля в данной точке.

Величина, численно равная отношению максимальной силы, действующей на проводник с током, на произведение силы тока и длины проводника, называется модулем вектора магнитной индукции:

Единицей измерения магнитной индукции является 1 тесла (Тл).

1Тл = 1Н/(1А∙1м).

Закон Ампера:

Сила, действующая на проводник с током в магнитном поле, равна произведению модуля магнитной индукции, силы тока, длины проводника и синуса угла между вектором магнитной индукции и направлением тока:

где α – угол между вектором B и направлением тока.

Направление силы Ампера определяется правилом левой руки:

Если ладонь левой руки развернуть так, чтобы линии магнитной индукции входили в ладонь, а четыре вытянутых пальца были направлены по направлению тока, то отогнутый на 90

0 большой палец покажет направление силы Ампера.

Сила Ампера – сила, действующая на проводник с током со стороны магнитного поля.

Сила Лоренца – сила, действующая на движущуюся заряженную частицу со стороны магнитного поля. Её численное значение равно произведению заряда частицы на модули скорости и магнитной индукции и синус угла меду векторами скорости и магнитной индукции:

– заряд частицы;

– скорость частицы;

B – модуль магнитной индукции;

– угол между векторами скорости частицы и магнитной индукции.

Направление силы Лоренца также определяют по правилу левой руки:

Если четыре вытянутых пальца левой руки направлены вдоль вектора скорости заряженной частицы, а вектор магнитной индукции направлен в ладонь, то отведённый на 900 большой палец покажет направление силы Лоренца. Если частица имеет заряд отрицательного знака, то направление силы Лоренца противоположно тому направлению, которое имела бы положительная частица.

Получим формулы для радиуса окружности и периода вращения частицы, которая влетает в однородное магнитное поле перпендикулярно линиям магнитной индукции, применяя формулы второго закона Ньютона и центростремительного ускорения.

Согласно 2-му закону Ньютона

Отсюда

Время, за которое частица делает полный оборот (период обращения), равно:

Многим юным бывает досадно, что они не родились в старые времена, когда делались открытия. Им кажется, что теперь всё известно и никаких открытий на их долю не осталось.

Одной из нераскрытых тайн является механизм земного магнитного поля. Как же и чем вызывается магнитное поле Земли? Подумайте и может быть…

Одна из возможных гипотез.

Как известно, ядро Земли имеет высокую температуру

и высокую плотность. Судя по исследованиям, в самом центре содержится твёрдое ядро. При вращении Земли вокруг своей оси центр тяжести не совпадает с геометрическим центром из-за притяжения Солнца. В результате сместившееся из центра ядро вращаясь относительно оболочки Земли вызывает такое же движение жидкой расплавленной массы мантии, как чайная ложка, перемешивающая воду в стакане. Получается не что иное, как направленное движение зарядов. Есть электрический ток, а он, в свою очередь, создаёт магнитное поле.

Разбор тренировочных заданий

1. На рисунке изображён проводник с током, помещённый в магнитное поле. Стрелка указывает направление тока в проводнике. Вектор магнитной индукции направлен перпендикулярно плоскости рисунка к нам. Как направлена сила, действующая на проводник с током?

Варианты ответов:

1. вправо →;

2. влево ←;

3. вниз ↓;

4. вверх ↑.

– точка означает, что магнитная индукция направлена на нас из глубины плоскости рисунка.

Используя правило левой руки, определяем направление силы Ампера:

Левую руку располагаем так, чтобы линии магнитной индукции входили в ладонь, 4 пальца направим вниз по направлению тока, тогда отогнутый на 900 большой палец покажет направление силы Ампера, т. е. она направлена влево.

Правильный вариант:

2. влево ←.

2. По проводнику длиной 40 см протекает ток силой 10 А. Чему равна индукция магнитного поля, в которое помещён проводник, если на проводник действует сила 8 мН?

(Ответ выразите в мТл).

3. Определите модуль силы, действующей на проводник длиной 50 см при силе тока 10 А в магнитном поле с индукцией 0,15 Тл. (Ответ выразите в мН).

4. Протон в магнитном поле с индукцией 0,01 Тл описал окружность радиусом 10 см. Найдите скорость протона. (Ответ выразите в км/с, округлив до десятков)

5. С какой скоростью влетает электрон в однородное магнитное поле (индукция 1,8 Тл) перпендикулярно к линиям индукции, если магнитное поле действует на него с силой 3,6∙10¹² Н? Ответ выразите в км/с.

6. Электрон движется в однородном магнитном поле с индукцией 3,14мТл. Чему равен период обращения электрона? (Ответ выразите в наносекундах, округлив до целых)

2. Дано:

l = 40cм = 0,4 м,

I = 10 A,

F =8 мН = 0,008 Н.

Найти: B

Решение:

Запишем формулу модуля магнитной индукции:

Делаем расчёт:

B = 0,008 Н / ( 0,4м·10 A) = 0,002 Tл = 2 мTл.

Ответ: 2 мTл.

3. Дано:

l = 50 cм = 0,5 м,

I = 10 A,

B = 0,l5 Tл.

Найти: F

Решение:

Запишем формулу силы Ампера:

Делаем расчёт:

F = 0,l5 Tл· 10 A· 0,5 м = 0,75 Н = 750 мН

Ответ: 750 мН.

4. Дано:

B = 0,0l Tл,

r = l0 cм = 0,l м.

Найти: v

Решение:

Заряд протона равен: q₀ = l,6·l0⁻ˡ⁹ Кл,

масса протона: m = l,67·l0⁻²⁷ кг.

Согласно 2-му закону Ньютона:

Отсюда следует:

Делаем расчёт:

v = ( l,6·l0⁻ˡ⁹ Кл·0,l м·0,0l Tл) / l,67·l0⁻²⁷ кг ≈ 0,00096·l0⁸ м/с ≈ l00 км/с.

Ответ: v ≈ l00 км/с.

5. Дано:

B = l,8 Tл,

F = 3,6·l0⁻¹² Н,

α = 90°.

Найти:

Решение:

Заряд электрона равен: q₀ = l,6·l0⁻ˡ⁹ Кл.

Используем формулу силы Лоренца:

.

Выразим из формулы силы скорость, учитывая, что sin90°=l,

Делаем расчёт:

v = 3,6·l0⁻¹² Н / (l,6·l0⁻ˡ⁹ Кл· l,8 Tл) = l,25·l0⁷м/с = l2500 км/с.

Ответ: v = l2500 км/с.

6. Дано:

B = 3,l4 мТл = 3,l4·l0⁻³ Tл,

q₀ = l,6·l0⁻ˡ⁹ Кл,

Найти: Т

Решение:

Масса электрона равна: m = 9,l·l0⁻³¹ кг.

Время, за которое частица делает полный оборот (период обращения), равно:

Делаем расчёт:

T = 2·3,l4·9,l·l0⁻³¹ кг/( l,6·l0⁻ˡ⁹ Кл·3,l4·l0⁻³ Tл) = ll,375·l0⁻⁹ с ≈ ll нс.

Ответ: T ≈ ll нс.

Магнитное поле. Силы – материалы для подготовки к ЕГЭ по Физике

Автор статьи — профессиональный репетитор, автор учебных пособий для подготовки к ЕГЭ Игорь Вячеславович Яковлев

Темы кодификатора ЕГЭ: сила Ампера, сила Лоренца.

В отличие от электрического поля, которое действует на любой заряд, магнитное поле действует только на движущиеся заряженные частицы. При этом оказывается, что сила зависит не только от величины, но и от направления скорости заряда.

Сила Лоренца

Сила, с которой магнитное поле действует на заряженную частицу, называется силой Лоренца. Опыт показывает, что вектор силы Лоренца находится следующим образом.

1. Абсолютная величина силы Лоренца равна:

(1)

Здесь — абсолютная величина заряда, — скорость заряда, — индукция магнитного поля, — угол между векторами и .

2. Сила Лоренца перпендикулярна обоим векторам и . Иными словами, вектор перпендикулярен плоскости, в которой лежат векторы скорости заряда и индукции магнитного поля.

Остаётся выяснить, в какое полупространство относительно данной плоскости направлена сила Лоренца.

3. Взаимное расположение векторов , и для положительного заряда показано на рис. 1.

Рис. 1. Сила Лоренца

Направление силы Лоренца определяется в данном случае по одному из двух альтернативных правил.

Правило часовой стрелки. Сила Лоренца направлена туда, глядя откуда кратчайший поворот вектора скорости частицы v к вектору магнитной индукции B виден против часовой стрелки.

Правило левой руки . Располагаем левую руку так, чтобы четыре пальца указывали направление скорости частицы, а линии поля входили в ладонь. Тогда оттопыренный большой палец укажет направление силы Лоренца.
Для отрицательного заряда направление силы Лоренца меняется на противоположное.

Всё вышеперечисленное является обобщением опытных фактов. Формула (1) позволяет связать размерность индукции магнитного поля с размерностями других физических величин:

Сила Ампера

Если металлический проводник с током поместить в магнитное поле, то на этот проводник со стороны магнитного поля будет действовать сила, которая называется силой Ампера.

Происхождение силы Ампера легко понять. Ведь ток в металле является направленным движением электронов, а на каждый электрон действует сила Лоренца. Все эти силы Лоренца, действующие на свободные электроны, имеют одинаковое направление и одинаковую величину; они складываются друг с другом и дают результирующую силу Ампера.

Направление силы Ампера определяется по тем же двум правилам, сформулированным выше.

Правило часовой стрелки . Сила Ампера направлена туда, глядя откуда кратчайший поворот тока к полю виден против часовой стрелки .

Правило левой руки . Располагаем левую руку так, чтобы четыре пальца указывали направление тока, а линии поля входили в ладонь. Тогда оттопыренный большой палец укажет направление силы Ампера .

Взаимное расположение тока, поля и силы Ампера указано на рис. 2.

Рис. 2. Сила Ампера

На этом рисунке проводник имеет длину , а угол между направлениями тока и поля равен . Мы сейчас выведем выражение для абсолютной величины силы Ампера.

На каждый свободный электрон действует сила Лоренца:

где — скорость направленного движения свободных электронов в проводнике.

Пусть — число свободных электронов в данном проводнике, — их концентрация (число в единице объёма). Тогда:

где — объём проводника, — площадь его поперечного сечения. Получаем:

Мы не случайно выделили скобками четыре сомножителя. Ведь это есть не что иное, как сила тока: (вспомните выражение силы тока через скорость направленного движения свободных зарядов!). В результате приходим к окончательной формуле для силы Ампера:

(2)

Хорошую возможность поупражняться в нахождении направлений магнитного поля и силы Ампера даёт взаимодействие параллельных токов. Оказывается, два параллельных провода отталкиваются, если направления токов в них противоположны, и притягиваются, если направления токов совпадают (рис. 3).

Рис. 3. Взаимодействие параллельных токов

Обязательно убедитесь в этом самостоятельно! Делаем так. Сначала берём произвольную точку на первом проводе и определяем направление магнитного поля, создаваемого в этой точке вторым проводом (правило вам известно — см. предыдущий листок>). Ну а затем находим направление силы Ампера, действующей на первый провод со стороны магнитного поля второго провода.

Рамка с током в магнитном поле

В листках по термодинамике мы говорили о важности циклически работающих машин: они снабжают нас энергией. Понимание законов термодинамики позволило сконструировать тепловые двигатели, которые исправно служат нам и по сей день.

Понимание же законов электромагнетизма дало возможность создать циклическую машину другого типа — электродвигатель.

Мы рассмотрим один из элементов электродвигателя — рамку с током в магнитном поле. Разобравшись в её поведении, мы сможем уловить основную идею функционирования электродвигателя.

Пусть прямоугольная рамка может вращаться вокруг горизонтальной оси (рис. 4, слева). Рамка находится в вертикальном однородном магнитном поле . Ток течёт по рамке в направлении ; это направление показано соответствующими стрелками.

Рис. 4. Рамка с током в магнитном поле

Вектор называется вектором нормали; он перпендикулярен плоскости рамки и направлен туда, глядя откуда ток кажется циркулирующим против часовой стрелки. (Иными словами, вектор сонаправлен с вектором индукции магнитного поля, которое создаётся током в рамке.) Поворот рамки измеряется углом между векторами и .

Теперь определим направления сил Ампера, которые действуют на рамку со стороны магнитного поля. Эти силы расставлены на рисунке; вот вам ещё одно упражнение на правило часовой стрелки (левой руки) — обязательно проверьте правильность указанных направлений!

Силы и , приложенные к сторонам и , действуют вдоль оси вращения. Они лишь растягивают рамку и не вызывают её вращение.

Куда более интересны силы и , приложеные соответственно к сторонам и . Они лежат в горизонтальной плоскости и перпендикулярны оси вращения. Эти силы вращают рамку в направлении по часовой стрелке, если смотреть справа (рис. 4, правая часть). Вычислим момент этой пары сил относительно оси вращения рамки.

Пусть длина стороны равна . Тогда

Пусть длина стороны равна . Плечо силы , как видно из рис. 4 (справа) равно:

Таким же будет плечо силы . Отсюда получаем момент сил, вращающий рамку:

Теперь заметим, что — площадь рамки. Окончательно имеем:

(3)

В этой формуле площадь служит единственной геометрической характеристикой рамки. Это наводит на мысль, что только площадь рамки и существенна в выражении для вращающего момента. И действительно, можно доказать (разбивая рамку на бесконечно узкие полоски, неотличимые от прямоугольников), что формула (3) справедлива для рамки любой формы с площадью .

Как видно из формулы (3), максимальный вращающий момент равен:

Эта максимальная величина момента достигается при , то есть когда плоскость рамки параллельна магнитному полю.

Вращающий момент становится равным нулю при и . Оба этих положения по-своему интересны.

При плоскость рамки перпендикулярна полю, а векторы и направлены в разные стороны. Данное положение является положением неустойчивого равновенсия: стоит хоть немного шевельнуть рамку, как силы Ампера начнут её вращать в том же направлении, поворачивая вектор к вектору (убедитесь!).

При плоскость рамки также перпендикулярна полю, а векторы и сонаправлены. Это — положение устойчивого равновенсия: при отклонении рамки возникает вращающий момент, стремящийся вернуть рамку назад (убедитесь!). Начнутся колебания рамки, постепенно затухающие из-за трения. В конце концов рамка остановится в положении ; в этом положении вектор индукции магнитного поля рамки сонаправлен с вектором индукции внешнего магнитного поля (вот почему при намагничивании вещества элементарные токи ориентируются так, что их поля направлены в сторону внешнего магнитного поля). Полезное сопоставление: рамка занимает такое положение, что её положительная нормаль ориентируется в том же направлении, что и северный конец стрелки компаса, помещённой в это магнитное поле.

Таким образом, поведение рамки в магнитном поле становится ясным: если отклонить рамку от положения устойчивого равновесия и отпустить, то рамка будет совершать колебания. С точки зрения совершения механической работы это не очень хорошо: если намотать нить на ось вращения и подвесить к нити груз, то груз будет то подниматься, то опускаться.
Но вот если исхитриться и заставить ток менять направление в нужные моменты, то вместо колебаний рамки начнётся её непрерывное вращение и, соответственно, непрерывный подъём подвешенного груза. Тогда-то и получится полноценный электродвигатель; идея с переменой направления тока реализуется с помощью коллектора и щёток.

Действие магнитного поля на проводник с током. Все о магнитах :: Класс!ная физика

  ДЕЙСТВИЕ МАГНИТНОГО ПОЛЯ НА ПРОВОДНИК С ТОКОМ


Магнитное  поле  действует  с  некоторой  силой  на  любой  проводник  с  током, 
находящийся  в  нем.
Если проводник, по которому протекает электрический ток подвесить в магнитном поле, например, между полюсами магнита, то магнитное поле будет действовать на проводник с некоторой силой и отклонять его.

Направление  движения  проводника 
зависит  от  направления  тока  в  проводнике  и  от   расположения  полюсов  магнита.

Действие  силы на рамку с током.

Если поместить проволочную рамку , по которой протекает электрический ток, в магнитное поле,
то в результате действия силы магнитного поля, рамка будет поворачиваться.

ЭЛЕКТРОДВИГАТЕЛЬ ПОСТОЯННОГО ТОКА



Устройство электродвигателя:
1) якорь  электродвигателя  –  железный  цилиндр, закрепленный  на  валу  двигателя;
вдоль  цилиндра  сделаны  прорези  (пазы ),  в  которые укладывается   обмотка, 
состоящая  из  большого  числа  витков  проволоки.
2) индуктор  электродвигателя – электромагнит;  образующий  магнитное  поле,
  в  котором  вращается  якорь  двигателя.

Принцип работы электродвигателя основан  на  вращении  катушки  с  током  в  магнитном  поле: магнитное  поле  создается  электромагнитом; 
катушка  –  обмотка  якоря,  по  которой  протекает  электрический  ток;  
со  стороны  магнитного  поля  на  катушку,  как  на  рамку  с  током  действует  сила, 
стремящаяся  повернуть  ее; 
  вместе  с  якорем   вращается  и  вал  двигателя.

Преимущества электродвигателей :

малые  размеры  по  сравнению  с  тепловыми  двигателями;
экологически  чистые;
можно  сделать любых  размеров;
высокий  КПД  (98   ).

ЧИТАЕМ !

“Поющие” магниты.
Магнитная летательная машина.
Электромагнитный транспорт.
Наподобие “магометова гроба”.

Тайны магнита.

Устали? – Отдыхаем!

Силы Ампера и Лоренца 10 класс онлайн-подготовка на Ростелеком Лицей

Силы Ампера и Лоренца

Магнитное поле действует не только на магниты, но и на движущиеся заряды. Здесь может быть два случая:

  1. Действие на движение отдельных зарядов в свободном пространстве.
  2. Действие на движение зарядов в проводнике.

1. Действие магнитного поля на отдельный заряд

Магнитное поле не действует на неподвижные заряды – только на движущиеся. Магнитное поле действует действует на заряды не прямолинейно, а всегда вбок.

Рассмотрим заряд, движущийся с некоторой скоростью. Если магнитное поле направленно вдоль этой скорости, то никакая сила со стороны магнитного поля не действует.

Сила появляется, если магнитное поле направленно перпендикулярно скорости частицы. Эта сила перпендикулярна и вектору индукции магнитного поля B ⃗, и скорости.

Сила, действующая на отдельный заряд со стороны магнитного поля:

Когда магнитное поле перпендикулярно скорости, сила тем больше, чем сильнее магнитное поле, больше заряд и больше его скорость:

F=qυB

Если магнитное поле направленно под углом, то разложим магнитное поле на перпендикулярную и продольную составляющие. И вспомним, что продольная составляющая не действует на частицу; действует только перпендикулярная составляющая. Т.е. в выражении для силы надо вместо B написать B. Если угол между магнитным полем и скоростью alpha, то можно это выражение переписать в виде:

FL=qυBsinα

Эта сила называется силой Лоренца.

Направление этой силы можно определить с помощью правила левой руки: 1. Приложить левую руку так, чтобы скорость была направленна вдоль четырех пальцев. 

2. Повернуть руку так, чтобы магнитное поле входило в ладонь.

 3. Оттопыренный под прямым углом большой палец укажет направление силы.

 

Это работает для положительного заряда. Если заряд отрицательный, то направление силы будет противоположным.

2. Действие магнитного поля на проводник с током

Предположим, что магнитное поле перпендикулярно проводнику. Ток – это движение заряженных частиц, поэтому их скорости в среднем направленны вдоль проводника. И на каждую из них действует магнитное поле. Поэтому на проводник будет действовать некоторая суммарная сила, называемая силой Ампера.

В общем случае выражение для силы:

FA = IBlsinα

По сути, сила Ампера – это макроскопическое проявление силы Лоренца. Поэтому полезно сравнить размерности выражений для этих сил. Если все записано правильно, они должны совпадать. Действительно, для обоих выражений мы получаем Кл⋅м/c⋅Тл.

 

Направление силы Ампера также определяется правилом левой руки. Четыре пальца направляются на этот раз по току, магнитное поле входит в ладонь, большой палец указывает направление силы.

В таблице с формулами описывающими создание поля зарядами и действие поля на заряды добавились две формулы.

Задача

Рейка с сопротивлением R, массы m и длины l может без трения скользить по двум направляющим, оставаясь при этом всегда перпендикулярным им. К направляющим подключен источник, создающий напряжение U. Система помещена в однородное магнитное поле с индукцией B, перпендикулярное плоскости рисунка. Рейку отпускают без начальной скорости. До какой скорости она разгонится, пройдя расстояние L? Сопротивлением направляющих пренебречь.

Решение

Перед нами система, которая может ускорять металлические объекты

Вспомним урок «закон сохранения энергии»

Такая система называется рельсотрон. В мирных целях, она может двигать транспорт, разгонять объекты до огромных скоростей и даже имитировать падение космических микрометеоритов.

электромагнетизм | Определение, уравнения и факты

Электромагнетизм , наука о заряде, а также о силах и полях, связанных с зарядом. Электричество и магнетизм – два аспекта электромагнетизма.

Британская викторина

Викторина “Все о физике”

Кто был первым ученым, проведшим эксперимент по управляемой цепной ядерной реакции? Какая единица измерения для циклов в секунду? Проверьте свою физическую хватку с помощью этой викторины.

Электричество и магнетизм долгое время считались отдельными силами. Только в 19 веке они стали рассматриваться как взаимосвязанные явления. В 1905 году специальная теория относительности Альберта Эйнштейна без сомнения установила, что оба аспекта являются аспектами одного общего явления. Однако на практике электрические и магнитные силы ведут себя по-разному и описываются разными уравнениями. Электрические силы создаются электрическими зарядами, находящимися в состоянии покоя или в движении.С другой стороны, магнитные силы создаются только движущимися зарядами и действуют исключительно на движущиеся заряды.

Электрические явления происходят даже в нейтральной материи, потому что силы действуют на отдельные заряженные составляющие. В частности, электрическая сила отвечает за большинство физических и химических свойств атомов и молекул. Он невероятно силен по сравнению с гравитацией. Например, отсутствие только одного электрона на каждый миллиард молекул у двух 70-килограммовых (154-фунтовых) людей, стоящих на расстоянии двух метров (двух ярдов) друг от друга, оттолкнет их с силой в 30 000 тонн.В более привычном масштабе электрические явления ответственны за молнии и гром, сопровождающие определенные штормы.

Электрические и магнитные силы могут быть обнаружены в областях, называемых электрическими и магнитными полями. Эти поля имеют фундаментальную природу и могут существовать в космосе вдали от заряда или тока, которые их породили. Примечательно, что электрические поля могут создавать магнитные поля и наоборот, независимо от любого внешнего заряда. Как обнаружил в своей работе английский физик Майкл Фарадей, изменяющееся магнитное поле создает электрическое поле, лежащее в основе производства электроэнергии.Напротив, изменяющееся электрическое поле создает магнитное поле, как пришел к выводу шотландский физик Джеймс Клерк Максвелл. Математические уравнения, сформулированные Максвеллом, включают световые и волновые явления в электромагнетизм. Он показал, что электрические и магнитные поля путешествуют вместе в пространстве как волны электромагнитного излучения, при этом изменяющиеся поля взаимно поддерживают друг друга. Примерами электромагнитных волн, распространяющихся в пространстве независимо от материи, являются радио- и телевизионные волны, микроволны, инфракрасные лучи, видимый свет, ультрафиолетовый свет, рентгеновские лучи и гамма-лучи.Все эти волны движутся с одинаковой скоростью, а именно скоростью света (примерно 300 000 километров или 186 000 миль в секунду). Они отличаются друг от друга только частотой, с которой колеблются их электрическое и магнитное поля.

Получите подписку Britannica Premium и получите доступ к эксклюзивному контенту. Подпишитесь сейчас

Уравнения Максвелла по-прежнему обеспечивают полное и элегантное описание электромагнетизма вплоть до субатомного масштаба, но не включая его. Однако в 20 веке интерпретация его работ расширилась.Специальная теория относительности Эйнштейна объединила электрические и магнитные поля в одно общее поле и ограничила скорость всей материи скоростью электромагнитного излучения. В конце 1960-х физики обнаружили, что у других сил в природе есть поля с математической структурой, подобной структуре электромагнитного поля. Эти другие силы представляют собой сильное взаимодействие, ответственное за энергию, выделяемую при ядерном синтезе, и слабое взаимодействие, наблюдаемое при радиоактивном распаде нестабильных атомных ядер.В частности, слабые и электромагнитные силы были объединены в общую силу, называемую электрослабой силой. Цель многих физиков объединить все фундаментальные силы, включая гравитацию, в одну великую единую теорию, до сих пор не достигнута.

Важным аспектом электромагнетизма является наука об электричестве, которая занимается поведением агрегатов заряда, включая распределение заряда в материи и движение заряда с места на место.Различные типы материалов классифицируются как проводники или изоляторы в зависимости от того, могут ли заряды свободно перемещаться через составляющие их вещества. Электрический ток – это мера потока зарядов; законы, управляющие токами в материи, важны в технологиях, особенно в производстве, распределении и контроле энергии.

Понятие напряжения, как и понятия заряда и тока, является фундаментальным в науке об электричестве. Напряжение – это мера склонности заряда перетекать из одного места в другое; положительные заряды обычно имеют тенденцию перемещаться из области высокого напряжения в область более низкого напряжения.Распространенная проблема в электричестве – это определение взаимосвязи между напряжением и током или зарядом в данной физической ситуации.

Эта статья стремится дать качественное понимание электромагнетизма, а также количественную оценку величин, связанных с электромагнитными явлениями.

Движение заряженной частицы в магнитном поле

Электрические силы против магнитных

И электрические, и магнитные силы влияют на траекторию заряженных частиц, но качественно по-разному.

Цели обучения

Сравните влияние электрического и магнитного полей на заряженную частицу

Основные выводы

Ключевые моменты
  • Сила, действующая на заряженную частицу из-за электрического поля, направлена ​​параллельно вектору электрического поля в случае положительного заряда и антипараллельно в случае отрицательного заряда. Это не зависит от скорости частицы.
  • Напротив, магнитная сила, действующая на заряженную частицу, ортогональна вектору магнитного поля и зависит от скорости частицы.Правило правой руки можно использовать для определения направления силы.
  • Электрическое поле может действовать на заряженную частицу, в то время как магнитное поле не действует.
  • Сила Лоренца – это комбинация электрической и магнитной сил, которые часто рассматриваются вместе в практических приложениях.
  • Линии электрического поля генерируются на положительных зарядах и оканчиваются на отрицательных. Силовые линии изолированного заряда направлены прямо радиально наружу. Электрическое поле касается этих линий.
  • Силовые линии магнитного поля в случае магнита генерируются на северном полюсе и заканчиваются на южном полюсе. Магнитные полюса не существуют изолированно. Как и в случае силовых линий электрического поля, магнитное поле касается силовых линий. Заряженные частицы будут вращаться вокруг этих силовых линий.
Ключевые термины
  • ортогональный : из двух объектов под прямым углом; перпендикулярны друг другу.

Электрические силы против магнитных

Сила, создаваемая как электрическими, так и магнитными силами, будет влиять на движение заряженных частиц.Однако результирующее изменение траектории частиц будет качественно отличаться между двумя силами. Ниже мы кратко рассмотрим два типа сил, а также сравним и сопоставим их влияние на заряженную частицу.

Электростатическая сила и магнитная сила на заряженной частице

Напомним, что в статическом неизменном электрическом поле E сила, действующая на частицу с зарядом q, будет:

[латекс] \ text {F} = \ text {qE} [/ латекс]

Где F – вектор силы, q – заряд, а E – вектор электрического поля.{2}} [/ латекс]

Следует подчеркнуть, что электрическая сила F действует параллельно электрическому полю E . Ротор электрической силы равен нулю, т.е .:

[латекс] \ bigtriangledown \ times \ text {E} = 0 [/ латекс]

Следствием этого является то, что электрическое поле может работать, и заряд в чистом электрическом поле будет следовать по касательной к линии электрического поля.

Напротив, напомним, что магнитная сила, действующая на заряженную частицу, ортогональна магнитному полю, так что:

[латекс] \ text {F} = \ text {qv} \ times \ text {B} = \ text {qvBsin} \ theta [/ latex]

, где B – вектор магнитного поля, v – скорость частицы, а θ – угол между магнитным полем и скоростью частицы.Направление F можно легко определить с помощью правила правой руки.

Правило правой руки : Магнитные поля действуют на движущиеся заряды. Эта сила – одна из самых основных известных. Направление магнитной силы на движущийся заряд перпендикулярно плоскости, образованной v и B, и следует правилу правой руки – 1 (RHR-1), как показано. Величина силы пропорциональна q, v, B и синусу угла между v и B.

Если скорость частицы выровнена параллельно магнитному полю или равна нулю, магнитная сила будет равна нулю.Это отличается от случая электрического поля, где скорость частицы не имеет никакого отношения в любой данный момент к величине или направлению электрической силы.

Угловая зависимость магнитного поля также заставляет заряженные частицы двигаться перпендикулярно линиям магнитного поля по кругу или по спирали, в то время как частица в электрическом поле будет двигаться по прямой линии вдоль линии электрического поля.

Еще одно различие между магнитными и электрическими силами состоит в том, что магнитные поля не работают, , поскольку движение частицы является круговым и, следовательно, заканчивается в одном и том же месте.Мы выражаем это математически как:

[латекс] \ text {W} = \ oint \ text {B} \ cdot \ text {dr} = 0 [/ latex]

Лоренц Форс

Сила Лоренца – это объединенная сила, действующая на заряженную частицу, вызванная как электрическим, так и магнитным полями, которые часто рассматриваются вместе для практических приложений. Если частица заряда q движется со скоростью v в присутствии электрического поля E и магнитного поля B , то на нее будет действовать сила:

[латекс] \ text {F} = \ text {q} [\ text {E} + \ text {vBsin} \ theta] [/ latex]

Линии электрического и магнитного поля

Выше мы вкратце упоминали, что движение заряженных частиц относительно силовых линий различается в зависимости от того, имеем ли мы дело с электрическими или магнитными полями.Есть некоторые заметные различия между концептуальными представлениями силовых линий электрического и магнитного поля. Линии электрического поля от положительного изолированного заряда представляют собой просто последовательность равномерно расположенных радиально направленных линий, направленных наружу от заряда. В случае отрицательного заряда направление поля меняется на противоположное. Электрическое поле направлено по касательной к силовым линиям. Конечно, мы предполагаем, что силовые линии тем плотнее упакованы, чем больше заряды. Хорошо видно, что ротор электрической силы равен нулю.

Электрическое поле, создаваемое точечными зарядами : электрическое поле, окружающее три различных точечных заряда: (а) положительный заряд; (б) отрицательный заряд равной величины; (c) больший отрицательный заряд.

Если задействовано несколько зарядов, силовые линии формируются на положительных зарядах и заканчиваются на отрицательных.

В случае магнитов силовые линии формируются на северном полюсе (+) и заканчиваются на южном полюсе (-) – см. Рисунок ниже.Однако магнитные «заряды» всегда идут парами – магнитных монополей (изолированных северных или южных полюсов) нет. Следовательно, ротор магнитного поля, создаваемого обычным магнитом, всегда отличен от нуля. Заряженные частицы будут вращаться по спирали вокруг этих силовых линий до тех пор, пока частицы имеют ненулевую составляющую скорости, направленную перпендикулярно силовым линиям.

Модель магнитного полюса : Модель магнитного полюса: два противоположных полюса, Северный (+) и Южный (-), разделенные расстоянием d, создают H-поле (линии).

Магнитное поле может также создаваться током, силовые линии которого представляют собой концентрические круги вокруг токоведущего провода. Магнитная сила в любой точке в этом случае может быть определена с помощью правила правой руки, и она будет перпендикулярна обоим. ток и магнитное поле.

При постоянной скорости получается прямая

Если скорость заряженной частицы параллельна магнитному полю, результирующая сила отсутствует и частица движется по прямой линии.

Цели обучения

Определить условия, при которых частица движется по прямой в магнитном поле

Основные выводы

Ключевые моменты
  • Первый закон движения Ньютона гласит, что если объект не испытывает чистой силы, то его скорость постоянна.
  • Частица с постоянной скоростью будет двигаться по прямой в пространстве.
  • Если скорость заряженной частицы полностью параллельна магнитному полю, магнитное поле не будет оказывать на частицу силы и, таким образом, скорость останется постоянной.
  • В случае, если вектор скорости не параллелен и не перпендикулярен магнитному полю, составляющая скорости, параллельная полю, останется постоянной.
Ключевые термины
  • прямолинейное движение : движение, которое происходит в одном направлении

Постоянная скорость обеспечивает прямолинейное движение

Вспомните первый закон движения Ньютона. Если объект не испытывает чистой силы, то его скорость постоянна: объект либо находится в состоянии покоя (если его скорость равна нулю), либо он движется по прямой с постоянной скоростью (если его скорость отлична от нуля).

Во многих случаях частица может не испытывать результирующей силы. Частица могла существовать в вакууме вдали от любых массивных тел (которые проявляют гравитационные силы) и электромагнитных полей. Или на частицу могут действовать две или более силы, уравновешенные таким образом, что результирующая сила равна нулю. Так обстоит дело, скажем, с частицей, подвешенной в электрическом поле, электрическая сила которого точно уравновешивает гравитацию.

Если результирующая сила, действующая на частицу, равна нулю, то ускорение обязательно равно нулю в соответствии со вторым законом Ньютона: F = ma.Если ускорение равно нулю, любая скорость частицы будет поддерживаться бесконечно (или до тех пор, пока результирующая сила не станет равной нулю). Поскольку скорость является вектором, направление остается неизменным вместе со скоростью, поэтому частица движется в одном направлении, например, по прямой.

Заряженные частицы, движущиеся параллельно магнитным полям

Сила, которую заряженная частица «ощущает» из-за магнитного поля, зависит от угла между вектором скорости и вектором магнитного поля B .Напомним, что магнитная сила составляет:

Нулевая сила, когда скорость параллельна магнитному полю : В приведенном выше случае магнитная сила равна нулю, потому что скорость параллельна силовым линиям магнитного поля.

[латекс] \ text {F} = \ text {qvBsin} \ theta [/ latex]

Если магнитное поле и скорость параллельны (или антипараллельны), то sinθ равен нулю и сила отсутствует. В этом случае заряженная частица может продолжать прямолинейное движение даже в сильном магнитном поле.Если находится в диапазоне от 0 до 90 градусов, то составляющая v , параллельная B , остается неизменной.

Круговое движение

Поскольку магнитная сила всегда перпендикулярна скорости заряженной частицы, частица будет совершать круговое движение.

Цели обучения

Опишите условия, которые приводят к круговому движению заряженной частицы в магнитном поле

Основные выводы

Ключевые моменты
  • Магнитное поле не работает, поэтому кинетическая энергия и скорость заряженной частицы в магнитном поле остаются постоянными.{2}} {\ text {r}} [/ latex].
  • Решение для r выше дает гриорадиус, или радиус кривизны траектории частицы с зарядом q и массой m, движущейся в магнитном поле с напряженностью B. Тогда гриорадиус определяется как [латекс] \ text {r} = \ frac {\ text {mv}} {\ text {qB}} [/ latex].
  • Циклотронная частота (или, что то же самое, гирочастота) – это количество циклов, которые частица совершает вокруг своего кругового цикла каждую секунду, и определяется как [latex] \ text {f} = \ frac {\ text {qB}} {2 \ пи \ текст {м}} [/ латекс].
Ключевые термины
  • гирорадиус : Радиус кругового движения заряженной частицы в присутствии однородного магнитного поля.
  • циклотронная частота : частота заряженной частицы, движущейся перпендикулярно направлению однородного магнитного поля B (постоянная величина и направление). Дается равенством центростремительной силы и магнитной силы Лоренца.

Круговое движение заряженной частицы в магнитном поле

Магнитные силы могут заставлять заряженные частицы двигаться по круговой или спиральной траектории.Ускорители элементарных частиц удерживают протоны на круговых траекториях с помощью магнитной силы. Космические лучи будут следовать по спирали при встрече с магнитным полем астрофизических объектов или планет (одним из примеров является магнитное поле Земли). На фотографии пузырьковой камеры на рисунке ниже показаны заряженные частицы, движущиеся по таким искривленным траекториям. Изогнутые траектории заряженных частиц в магнитных полях являются основой ряда явлений и могут даже использоваться аналитически, например, в масс-спектрометре.показывает путь, пройденный частицами в пузырьковой камере.

Пузырьковая камера : Следы пузырьков создаются заряженными частицами высокой энергии, движущимися через перегретый жидкий водород в изображении пузырьковой камеры этим художником. Существует сильное магнитное поле, перпендикулярное странице, которое вызывает искривленные траектории частиц. Радиус пути можно использовать для определения массы, заряда и энергии частицы.

Итак, вызывает ли магнитная сила круговое движение? Магнитная сила всегда перпендикулярна скорости, поэтому она не действует на заряженную частицу.Таким образом, кинетическая энергия и скорость частицы остаются постоянными. Это влияет на направление движения, но не на скорость. Это типично для равномерного кругового движения. Самый простой случай возникает, когда заряженная частица движется перпендикулярно однородному B-полю, как показано на рисунке. (Если это происходит в вакууме, магнитное поле является доминирующим фактором, определяющим движение.) Здесь магнитная сила (Лоренц сила) обеспечивает центростремительную силу

Круговое движение заряженной частицы в магнитном поле : отрицательно заряженная частица движется в плоскости страницы в области, где магнитное поле перпендикулярно странице (представлено маленькими кружками с крестиками – как хвосты стрелок) .{2}} {\ text {r}} [/ latex]

решение относительно r дает

[латекс] \ text {r} = \ frac {\ text {mv}} {\ text {qB}} [/ latex]

Здесь r , называемый гирорадиусом или циклотронным радиусом, представляет собой радиус кривизны пути заряженной частицы с массой m и зарядом q , движущейся со скоростью v перпендикулярно магнитному полю прочность B . Другими словами, это радиус кругового движения заряженной частицы в присутствии однородного магнитного поля.Если скорость не перпендикулярна магнитному полю, то v является составляющей скорости, перпендикулярной полю. Компонент скорости, параллельный полю, не изменяется, поскольку магнитная сила равна нулю для движения, параллельного полю. Мы рассмотрим последствия этого случая в следующем разделе, посвященном спиральному движению.

Частица, совершающая круговое движение из-за однородного магнитного поля, называется циклотроном , резонанс .Этот термин происходит от названия циклотронного ускорителя частиц, показанного на рисунке. Циклотронная частота (или, что эквивалентно, гирочастота) – это количество циклов, которые частица совершает вокруг своего кругового контура каждую секунду, и может быть найдена путем решения для v выше и подставив частоту обращения так, чтобы

Циклотрон : Французский циклотрон, произведен в Цюрихе, Швейцария, в 1937 г.

[латекс] \ text {f} = \ frac {\ text {v}} {2 \ pi \ text {r}} [/ latex]

становится

[латекс] \ text {f} = \ frac {\ text {qB}} {2 \ pi \ text {m}} [/ latex]

Циклотронная частота тривиально выражается в радианах в секунду как

.

[латекс] \ omega = \ frac {\ text {qB}} {\ text {m}} [/ latex].

Спиральное движение

Винтовое движение возникает, когда вектор скорости не перпендикулярен вектору магнитного поля.

Цели обучения

Опишите условия, которые приводят к спиральному движению заряженной частицы в магнитном поле

Основные выводы

Ключевые моменты
  • Ранее мы видели, что круговое движение возникает, когда скорость заряженной частицы перпендикулярна магнитному полю. Скорость и кинетическая энергия частицы остаются постоянными, но направление изменяется в каждый момент перпендикулярной магнитной силой.
  • Если скорость не перпендикулярна магнитному полю, мы учитываем только компонент v, который перпендикулярен полю при проведении наших расчетов.
  • Компонент скорости, параллельный полю, не изменяется, поскольку магнитная сила равна нулю для движения, параллельного полю. Это вызывает спиральное движение.
  • Заряды могут двигаться по спирали вдоль силовых линий. Если сила магнитного поля увеличивается в направлении движения, поле будет оказывать силу, замедляющую заряды и даже меняющую их направление.Это называется магнитным зеркалом.
Ключевые термины
  • спиральное движение : движение, которое создается, когда одна составляющая скорости постоянна по величине и направлению (т. Е. Прямолинейное движение), в то время как другая составляющая постоянна по скорости, но равномерно изменяется по направлению (т. Е. Круговое движение. ). Это суперпозиция прямолинейного и кругового движения.
  • магнитное зеркало : конфигурация магнитного поля, при которой сила поля изменяется при движении вдоль силовой линии.Зеркальный эффект приводит к тенденции заряженных частиц отскакивать от области сильного поля.

Спиральное движение

В разделе о круговом движении мы описали движение заряженной частицы с вектором магнитного поля, перпендикулярным скорости частицы. В этом случае магнитная сила также перпендикулярна скорости (и, конечно, вектору магнитного поля) в любой данный момент, что приводит к круговому движению. Скорость и кинетическая энергия частицы остаются постоянными, но направление изменяется в каждый момент перпендикулярной магнитной силой.быстро рассматривает эту ситуацию в случае отрицательно заряженной частицы в магнитном поле, направленном внутрь страницы.

Круговое движение заряженной частицы в магнитном поле : отрицательно заряженная частица движется в плоскости страницы в области, где магнитное поле перпендикулярно странице (представлено маленькими кружками с крестиками – как хвосты стрелок) . Магнитная сила перпендикулярна скорости, поэтому скорость изменяется по направлению, но не по величине.2} {\ text {r}} [/ latex]

[латекс] \ text {F} = \ text {qvBsin} \ theta = \ text {qv} _ {\ perp} \ text {B} [/ latex]

Компонент скорости, параллельный полю, не изменяется, поскольку магнитная сила равна нулю для движения, параллельного полю. Это производит спиральное движение (т.е. спиральное движение), а не круговое движение.

показывает, как электроны, движущиеся не перпендикулярно силовым линиям магнитного поля, следуют за силовыми линиями. Компонент скорости, параллельный линиям, не изменяется, поэтому заряды вращаются по спирали вдоль силовых линий.Если напряженность поля увеличивается в направлении движения, поле будет оказывать силу, замедляющую заряды (и даже меняющую их направление), образуя своего рода магнитное зеркало.

Спиральное движение и магнитные зеркала : Когда заряженная частица движется вдоль силовой линии магнитного поля в область, где поле становится сильнее, частица испытывает силу, которая уменьшает составляющую скорости, параллельную полю. Эта сила замедляет движение вдоль силовой линии и переворачивает его, образуя «магнитное зеркало».«

Движение заряженных частиц в магнитных полях связано с такими разными вещами, как северное сияние или австралийское сияние (северное и южное сияние) и ускорители частиц. Заряженные частицы, приближающиеся к линиям магнитного поля, могут быть захвачены спиральными орбитами вокруг линий, а не пересекать их. , как показано выше. Некоторые космические лучи, например, следуют за линиями магнитного поля Земли, проникая в атмосферу вблизи магнитных полюсов и вызывая южное или северное сияние за счет ионизации молекул в атмосфере.Те частицы, которые приближаются к средним широтам, должны пересекать силовые линии магнитного поля, и многие из них не могут проникнуть в атмосферу. Космические лучи являются составной частью радиационного фона; следовательно, они дают более высокую дозу излучения на полюсах, чем на экваторе.

Заряженные частицы движутся по спирали вдоль линий магнитного поля Земли : Энергичные электроны и протоны, составляющие космических лучей, исходящие от Солнца и дальнего космоса, часто следуют за линиями магнитного поля Земли, а не пересекают их.(Напомним, что северный магнитный полюс Земли на самом деле является южным полюсом в смысле стержневого магнита.)

Примеры и приложения

Циклотроны, магнетроны и масс-спектрометры представляют собой практические технологические приложения электромагнитных полей.

Цели обучения

Обсудить применение масс-спектрометров, движение заряженных частиц в циклотроне и то, как микроволны генерируются в магнетроне с резонатором.

Основные выводы

Ключевые моменты
  • Циклотрон – это тип ускорителя частиц, в котором заряженные частицы ускоряются наружу от центра по спиральной траектории.Частицы удерживаются на спиральной траектории статическим магнитным полем и ускоряются быстро меняющимся электрическим полем.
  • Магнетрон с резонатором представляет собой мощную вакуумную лампу, которая генерирует микроволны, используя взаимодействие потока электронов с магнитным полем. Магнетрон находит применение в радарах, обогреве и освещении.
  • Масс-спектрометры измеряют отношение массы к заряду заряженных частиц с помощью электромагнитных полей для разделения частиц с разными массами и / или зарядами.Его можно использовать для определения элементного состава молекулы или образца.
Ключевые термины
  • циклотрон : ускоритель ранних частиц, в котором заряженные частицы генерировались в центральном источнике и ускорялись по спирали наружу через фиксированные магнитные и переменные электрические поля.
  • масс-спектрометр : устройство, используемое в масс-спектрометрии для определения массового состава данного вещества.
  • магнетрон : устройство, в котором электроны заставляют резонировать в камере особой формы и, таким образом, производить микроволновое излучение; используется в радарах и микроволновых печах.

Примеры и приложения – Движение заряженной частицы в магнитном поле

Обзор

Напомним, что заряженные частицы в магнитном поле будут двигаться по круговой или спиральной траектории в зависимости от совмещения их вектора скорости с вектором магнитного поля. Последствия такого движения могут иметь глубокое практическое применение. Многие технологии основаны на движении заряженных частиц в электромагнитных полях. Мы рассмотрим некоторые из них, включая циклотрон и синхротрон, магнетрон с резонатором и масс-спектрометр.

Циклотроны и синхротроны

Циклотрон – это тип ускорителя частиц, в котором заряженные частицы ускоряются наружу от центра по спиральной траектории. Частицы удерживаются на спиральной траектории статическим магнитным полем и ускоряются быстро меняющимся (радиочастотным) электрическим полем.

Cyclotron Sketch : Рисунок частицы, ускоряемой в циклотроне и выбрасываемой через канал.

Циклотроны ускоряют пучки заряженных частиц с помощью высокочастотного переменного напряжения, которое прикладывают между двумя электродами в форме буквы «D» (также называемыми «деээ»).Дополнительное статическое магнитное поле прикладывается перпендикулярно плоскости электрода, позволяя частицам повторно сталкиваться с ускоряющим напряжением много раз в одной и той же фазе. Для этого частота напряжения должна соответствовать частоте циклотронного резонанса частицы,

.

[латекс] \ text {f} = \ frac {\ text {qB}} {2 \ pi \ text {m}} [/ latex]

с релятивистской массой м и его зарядом q . Эта частота задается равенством центростремительной силы и магнитной силы Лоренца.Частицы, инжектируемые около центра магнитного поля, увеличивают свою кинетическую энергию только при рециркуляции через зазор между электродами; таким образом, они движутся наружу по спирали. Их радиус будет увеличиваться до тех пор, пока частицы не попадут в цель по периметру вакуумной камеры или не покинут циклотрон с помощью лучевой трубки, что позволит их использовать. Частицы, ускоренные циклотроном, можно использовать в терапии частицами для лечения некоторых видов рака. Кроме того, циклотроны являются хорошим источником пучков высоких энергий для ядерно-физических экспериментов.

Синхротрон является усовершенствованием циклотрона, в котором ведущее магнитное поле (изгибание частиц по замкнутому пути) зависит от времени, синхронизировано с пучком частиц с увеличивающейся кинетической энергией. Синхротрон – одна из первых концепций ускорителей, которые позволяют создавать крупномасштабные объекты, поскольку изгиб, фокусировка пучка и ускорение могут быть разделены на разные компоненты.

Полостной магнетрон

Магнетрон с резонатором представляет собой мощную вакуумную лампу, которая генерирует микроволны, используя взаимодействие потока электронов с магнитным полем.Все магнетроны с резонатором состоят из горячего катода с высоким (непрерывным или импульсным) отрицательным потенциалом, создаваемым высоковольтным источником постоянного тока. Катод встроен в центр вакуумированной лопастной круглой камеры. Магнитное поле, параллельное нити накала, создается постоянным магнитом. Магнитное поле заставляет электроны, притянутые к (относительно) положительной внешней части камеры, двигаться по спирали наружу по круговой траектории, что является следствием силы Лоренца. По краю камеры расположены цилиндрические полости.Полости открыты по своей длине и соединяют общее пространство полости. Проходя мимо этих отверстий, электроны создают резонансное высокочастотное радиополе в полости, которое, в свою очередь, заставляет электроны группироваться в группы.

Схема магнетрона с резонатором : Сечение магнетрона с резонансным резонатором. Магнитные силовые линии параллельны геометрической оси этой конструкции.

Размеры полостей определяют резонансную частоту и, следовательно, частоту излучаемых микроволн.Магнетрон – это автоколебательное устройство, не требующее никаких внешних элементов, кроме источника питания. Магнетрон находит практическое применение в радарах, обогреве (как основной компонент микроволновой печи) и освещении.

Масс-спектрометрия

Масс-спектрометрия – это аналитический метод измерения отношения массы к заряду заряженных частиц. Он используется для определения массы частиц и определения элементного состава образца или молекулы.

Масс-анализаторы разделяют ионы в соответствии с их отношением массы к заряду.Следующие два закона управляют динамикой заряженных частиц в электрическом и магнитном полях в вакууме:

[латекс] \ text {F} = \ text {Q} (\ text {E} + \ text {v} \ times \ text {B}) [/ latex] (сила Лоренца)

[латекс] \ text {F} = \ text {ma} [/ latex]

Приравнивая приведенные выше выражения для силы, приложенной к иону, получаем:

[латекс] (\ text {m} / \ text {Q}) \ text {a} = \ text {E} + \ text {v} \ times \ text {B} [/ latex]

Это дифференциальное уравнение вместе с начальными условиями полностью определяет движение заряженной частицы в терминах m / Q.Есть много типов масс-анализаторов, использующих статические или динамические поля, а также магнитные или электрические поля, но все они работают в соответствии с приведенным выше дифференциальным уравнением.

На следующем рисунке показан один тип масс-спектрометра. Отклонения частиц зависят от отношения массы к заряду. В случае изотопного диоксида углерода каждая молекула имеет одинаковый заряд, но разные массы. Масс-спектрометр будет разделять частицы в пространстве, позволяя детектору измерять отношение массы к заряду каждой частицы.Поскольку заряд известен, абсолютную массу можно определить тривиально. Относительные содержания могут быть выведены путем подсчета количества частиц каждой данной массы.

Масс-спектрометрия : Схема простого масс-спектрометра с масс-анализатором секторного типа. Он предназначен для измерения соотношения изотопов диоксида углерода (IRMS), как в дыхательном тесте с мочевиной углерода-13.

электромагнетизм – Магнитная сила, действующая на заряженную частицу, не влияет на общую энергию

Магнитная сила, действующая на заряженную частицу, не влияет на ее энергию.В противном случае магнитные силы не могут работать. Это потому, что уравнение магнитной силы задается

$$ \ vec {F} = q \ vec {v} \ times \ vec {B} $$

где $ q $ – заряд, $ \ vec {v} $ – скорость, а $ \ vec {B} $ – плотность магнитного потока.

Итак, ясно, что в силу члена перекрестного произведения магнитная сила, действующая на заряд, будет перпендикулярна плоскости, содержащей векторы $ \ vec {v} $ и $ \ vec {B} $. Если заряд входит перпендикулярно полю, то магнитная сила будет максимальной и ее величина будет $ F = qvB $.Это означает, что сила действует в направлении, перпендикулярном движению заряда (или вектору скорости $ \ vec {v} $). Это означает, что смещение и сила в разных направлениях. Таким образом, работа, совершаемая магнитной силой при смещении заряда, будет равна нулю, поскольку считается, что работа совершается, если смещение происходит в том же направлении, что и приложенная сила. Это означает, что магнитная сила не имеет ничего общего со скоростью частицы, поскольку смещение ($ \ vec {v} dt $) не создается магнитным полем .0 = 0 $). Следовательно, магнитная сила не действует на заряд . это означает, что кинетическая энергия заряда не увеличилась и не уменьшилась.

Но магнитная сила может ускорить заряженную частицу, отклоняя ее путь, не влияя на ее скорость $ v $. Итак, заряженная частица движется по круговой траектории. Помните, что у вас не может быть равномерного движения по круговой траектории. Даже если скорость постоянна, направление скорости меняется, что вызывает ускорение.

Таким образом, магнитная сила просто отклоняет заряд по круговой траектории и обеспечивает необходимую центростремительную силу, соответствующую величине поля. Вот почему магнитное поле не влияет на полную энергию заряженной частицы. Все меняется, если у вас есть электрическое поле.

электромагнетизм – Как движущиеся заряды создают магнитные поля?

Если вы плохо знакомы со специальной теорией относительности, нет способа по-настоящему объяснить это явление.Лучшее, что можно сделать, – это дать вам правил , пропитанных эзотерическими идеями, такими как «электромагнитное поле» и «лоренц-инвариантность». Конечно, это не то, что вам нужно, и это правильно, поскольку физика никогда не должна сводиться к безосновательному принятию правил, установленных свыше.

Дело в том, что магнетизм – это не что иное, как электростатика в сочетании со специальной теорией . К сожалению, вы не найдете много книг, объясняющих это – либо авторы ошибочно полагают, что уравнения Максвелла не имеют оправдания и должны приниматься на веру, либо они слишком погрязли в своих собственных эзотерических обозначениях, чтобы остановиться, чтобы задуматься о том, что они говорят.Единственная известная мне книга, которая правильно трактует эту тему, – это книга Перселла «Электричество и магнетизм », которая недавно была переиздана в третьем издании. (Второе издание отлично подойдет, если вы сможете найти копию.)

Краткий эвристический набросок идеи заключается в следующем. Предположим, есть линия положительных зарядов, движущаяся по оси $ z $ в положительном направлении – ток. Рассмотрим положительный заряд $ q $, расположенный в $ (x, y, z) = (1,0,0) $, движущийся в отрицательном направлении $ z $.Мы видим, что из-за всех этих зарядов на $ q $ будет некоторая электростатическая сила.

Но давайте попробуем что-нибудь безумное – давайте перейдем к системе отсчета $ q $. В конце концов, законы физики должны соблюдаться со всех точек зрения. Ясно, что заряды, составляющие ток, в этой системе координат будут двигаться быстрее. Но это мало что дает, поскольку в конце концов кулоновская сила явно не заботится о скорости зарядов, а только об их разделении. Но специальная теория относительности говорит нам о другом.2} $, где $ v $ – скорость $ q $ в исходном кадре. Это знаменитое сокращение длины, предсказанное специальной теорией относительности.

Если текущие заряды окажутся ближе друг к другу, тогда очевидно, что $ q $ почувствует большую электростатическую силу от оси $ z $ в целом. Он будет испытывать дополнительную силу в положительном направлении $ x $, от оси, сверх того, что мы ожидали, просто сидя в лабораторной раме. По сути, закон Кулона – это закон силы только , действующий на заряд, но только система покоя заряда действительна для использования этого закона для определения силы, которую испытывает заряд.

Вместо того, чтобы постоянно переключаться между кадрами, мы изобретаем магнитное поле как математическое устройство, которое выполняет то же самое. При правильном определении он будет полностью объяснять эту аномальную силу, которая, по-видимому, испытывает заряд, когда мы наблюдаем его не в его собственной системе покоя. В примере, который я только что рассмотрел, правило правой руки говорит вам, что мы должны приписать магнитное поле току, вращающемуся вокруг оси $ z $ таким образом, чтобы он указывал в положительном направлении $ y $ в месте расположения $ q $.Скорость заряда находится в отрицательном направлении $ z $, и поэтому $ q \ vec {v} \ times \ vec {B} $ указывает в положительном направлении $ x $, как мы узнали из изменения системы отсчета. .

Что такое магнетизм? | Магнитные поля и магнитная сила

Магнетизм – это один из аспектов комбинированной электромагнитной силы. Это относится к физическим явлениям, возникающим из-за силы, вызванной магнитами, объектами, которые создают поля, которые притягивают или отталкивают другие объекты.

Согласно веб-сайту HyperPhysics Университета штата Джорджия, магнитное поле действует на частицы в поле за счет силы Лоренца.Движение электрически заряженных частиц порождает магнетизм. Сила, действующая на электрически заряженную частицу в магнитном поле, зависит от величины заряда, скорости частицы и силы магнитного поля.

Все материалы обладают магнетизмом, некоторые сильнее, чем другие. Постоянные магниты, сделанные из таких материалов, как железо, испытывают сильнейшее воздействие, известное как ферромагнетизм. За редким исключением, это единственная форма магнетизма, достаточно сильная, чтобы ее могли почувствовать люди.

Противоположности притягиваются

Магнитные поля генерируются вращающимися электрическими зарядами, согласно HyperPhysics. Все электроны обладают свойством углового момента или спина. Большинство электронов имеют тенденцию образовывать пары, в которых один из них имеет «спин вверх», а другой – «спин вниз», в соответствии с принципом исключения Паули, который гласит, что два электрона не могут находиться в одном и том же энергетическом состоянии одновременно. В этом случае их магнитные поля направлены в противоположные стороны, поэтому они компенсируют друг друга.Однако некоторые атомы содержат один или несколько неспаренных электронов, спин которых может создавать направленное магнитное поле. По данным Ресурсного центра неразрушающего контроля (NDT), направление их вращения определяет направление магнитного поля. Когда значительное большинство неспаренных электронов выровнены со своими спинами в одном направлении, они объединяются, чтобы создать магнитное поле, достаточно сильное, чтобы его можно было почувствовать в макроскопическом масштабе.

Источники магнитного поля дипольные, с северным и южным магнитными полюсами.По словам Джозефа Беккера из Университета Сан-Хосе, противоположные полюса (северный и южный) притягиваются, а подобные полюса (северный и северный, или южный и южный) отталкиваются. Это создает тороидальное поле или поле в форме пончика, поскольку направление поля распространяется наружу от северного полюса и входит через южный полюс.

Земля сама по себе является гигантским магнитом. Согласно HyperPhysics, планета получает свое магнитное поле от циркулирующих электрических токов внутри расплавленного металлического ядра. Компас указывает на север, потому что маленькая магнитная стрелка в нем подвешена, так что он может свободно вращаться внутри своего корпуса, выравниваясь с магнитным полем планеты.Как ни парадоксально, то, что мы называем Северным магнитным полюсом, на самом деле является южным магнитным полюсом, потому что он притягивает северные магнитные полюса стрелок компаса.

Ферромагнетизм

Если выравнивание неспаренных электронов сохраняется без приложения внешнего магнитного поля или электрического тока, образуется постоянный магнит. Постоянные магниты – результат ферромагнетизма. Приставка «ферро» относится к железу, потому что постоянный магнетизм впервые наблюдался в форме естественной железной руды, называемой магнетитом, Fe 3 O 4 .Кусочки магнетита можно найти разбросанными на поверхности земли или вблизи нее, и иногда они намагничиваются. Эти встречающиеся в природе магниты называются магнитными камнями. «Мы до сих пор не уверены в их происхождении, но большинство ученых считают, что магнитный камень – это магнетит, в который попала молния», – говорится в сообщении Университета Аризоны.

Вскоре люди узнали, что можно намагнитить железную иглу, поглаживая ее магнитом, в результате чего большинство неспаренных электронов в игле выстраиваются в одном направлении.По данным НАСА, примерно в 1000 году нашей эры китайцы обнаружили, что магнит, плавающий в чаше с водой, всегда выстраивался в направлении север-юг. Таким образом, магнитный компас стал огромным помощником в навигации, особенно днем ​​и ночью, когда звезды были скрыты облаками.

Было обнаружено, что другие металлы, помимо железа, обладают ферромагнитными свойствами. К ним относятся никель, кобальт и некоторые редкоземельные металлы, такие как самарий или неодим, которые используются для создания сверхпрочных постоянных магнитов.

Другие формы магнетизма

Магнетизм принимает многие другие формы, но, за исключением ферромагнетизма, они обычно слишком слабы, чтобы их можно было наблюдать за исключением чувствительных лабораторных приборов или при очень низких температурах. Диамагнетизм был впервые открыт в 1778 году Антоном Бругнамсом, который использовал постоянные магниты в поисках материалов, содержащих железо. По словам Джеральда Кюстлера, широко публикуемого независимого немецкого исследователя и изобретателя, в его статье «Диамагнитная левитация – исторические вехи», опубликованной в Румынском журнале технических наук, Бругнамс заметил: «Только темный и почти фиолетовый висмут проявлял конкретное явление в исследовании; когда я положил его кусок на круглый лист бумаги, плавающий на воде, он оттолкнулся обоими полюсами магнита.

Было установлено, что висмут обладает самым сильным диамагнетизмом из всех элементов, но, как обнаружил Майкл Фарадей в 1845 году, это свойство всей материи отталкиваться магнитным полем.

Диамагнетизм вызван орбитальным движением электронов, создающих крошечные токовые петли, которые создают слабые магнитные поля, согласно HyperPhysics. Когда к материалу прикладывается внешнее магнитное поле, эти токовые петли имеют тенденцию выравниваться таким образом, чтобы противостоять приложенному полю.Это заставляет все материалы отталкиваться постоянным магнитом; однако результирующая сила обычно слишком мала, чтобы быть заметной. Однако есть некоторые заметные исключения.

Пиролитический углерод, вещество, похожее на графит, демонстрирует даже более сильный диамагнетизм, чем висмут, хотя и только вдоль одной оси, и фактически может подниматься над сверхсильным редкоземельным магнитом. Некоторые сверхпроводящие материалы демонстрируют еще более сильный диамагнетизм ниже своей критической температуры, поэтому над ними можно левитировать редкоземельные магниты.(Теоретически из-за их взаимного отталкивания один может левитировать над другим.)

Парамагнетизм возникает, когда материал временно становится магнитным при помещении в магнитное поле и возвращается в свое немагнитное состояние, как только внешнее поле удаляется. При приложении магнитного поля некоторые из неспаренных электронных спинов выравниваются с полем и преодолевают противоположную силу, создаваемую диамагнетизмом. Однако, по словам Дэниела Марша, профессора физики Южного государственного университета Миссури, эффект заметен только при очень низких температурах.

Другие, более сложные формы включают антиферромагнетизм, при котором магнитные поля атомов или молекул выстраиваются рядом друг с другом; и поведение спинового стекла, которое включает как ферромагнитные, так и антиферромагнитные взаимодействия. Кроме того, ферримагнетизм можно рассматривать как комбинацию ферромагнетизма и антиферромагнетизма из-за многих общих черт между ними, но, по данным Калифорнийского университета в Дэвисе, он все же имеет свою уникальность.

Электромагнетизм

Когда провод перемещается в магнитном поле, поле индуцирует в проводе ток.И наоборот, магнитное поле создается движущимся электрическим зарядом. Это соответствует закону индукции Фарадея, который лежит в основе электромагнитов, электродвигателей и генераторов. Заряд, движущийся по прямой линии, как по прямому проводу, создает магнитное поле, которое вращается вокруг провода по спирали. Когда этот провод превращается в петлю, поле приобретает форму пончика или тора. Согласно Руководству по магнитной записи (Springer, 1998) Marvin Cameras, это магнитное поле можно значительно усилить, поместив ферромагнитный металлический сердечник внутрь катушки.

В некоторых приложениях постоянный ток используется для создания постоянного поля в одном направлении, которое можно включать и выключать вместе с током. Это поле может затем отклонить подвижный железный рычаг, вызывая слышимый щелчок. Это основа телеграфа, изобретенного в 1830-х годах Сэмюэлем Ф. Б. Морсом, который позволял осуществлять связь на большие расстояния по проводам с использованием двоичного кода, основанного на импульсах большой и малой длительности. Импульсы посылались опытными операторами, которые быстро включали и выключали ток с помощью подпружиненного переключателя с мгновенным контактом или ключа.Другой оператор на принимающей стороне затем переводил слышимые щелчки обратно в буквы и слова.

Катушка вокруг магнита также может двигаться по шаблону с изменяющейся частотой и амплитудой, чтобы индуцировать ток в катушке. Это основа для ряда устройств, в первую очередь для микрофона. Звук заставляет диафрагму двигаться внутрь и наружу с волнами переменного давления. Если диафрагма соединена с подвижной магнитной катушкой вокруг магнитопровода, она будет производить переменный ток, аналогичный падающим звуковым волнам.Затем этот электрический сигнал может быть усилен, записан или передан по желанию. Крошечные сверхсильные магниты из редкоземельных металлов теперь используются для изготовления миниатюрных микрофонов для сотовых телефонов, сообщил Марш Live Science.

Когда этот модулированный электрический сигнал подается на катушку, он создает колеблющееся магнитное поле, которое заставляет катушку входить и выходить по магнитному сердечнику по той же схеме. Затем катушка прикрепляется к подвижному диффузору динамика, чтобы он мог воспроизводить слышимые звуковые волны в воздухе.Первым практическим применением микрофона и динамика был телефон, запатентованный Александром Грэмом Беллом в 1876 году. Хотя эта технология была улучшена и усовершенствована, она все еще является основой для записи и воспроизведения звука.

Применение электромагнитов практически бесчисленное множество. Закон индукции Фарадея формирует основу для многих аспектов нашего современного общества, включая не только электродвигатели и генераторы, но и электромагниты всех размеров. Тот же принцип, который используется гигантским краном для подъема старых автомобилей на свалку металлолома, также используется для выравнивания микроскопических магнитных частиц на жестком диске компьютера для хранения двоичных данных, и каждый день разрабатываются новые приложения.

Штатный писатель Таня Льюис внесла свой вклад в этот отчет.

Дополнительные ресурсы

SA: AP Physics 2 Избранный вопрос: заряженная частица в магнитном поле

Вопрос

Рассмотрим заряженную частицу, движущуюся в магнитном поле, которое не обязательно является однородным. Частица движется по пути, который не всегда параллелен направлению магнитного поля. Магнитная сила – единственная сила, которая действует на частицу.

(a) В этой части предположим, что частица не теряет энергию из-за электромагнитных воздействий из-за эффектов ускорения и что заряд движется с нерелятивистской начальной скоростью.

  1. Изменится направление движения частицы, но не ее скорость. Это происходит независимо от его собственного заряда или движения, силы или ориентации магнитного поля. Кратко объясните, почему это так.
  2. Если магнитное поле однородно как по силе, так и по направлению, а начальное движение частицы перпендикулярно направлению магнитного поля, частица будет двигаться по круговой траектории.Объясните, почему это происходит.

(б) Частица изначально расположена в точке P в плоскости страницы и движется вправо со скоростью v . Магнитное поле, через которое движется частица, создается длинным прямым проводом, по которому течет ток I вправо, расположенным в плоскости страницы под точкой P .

  1. Знак заряда положительный или отрицательный? Объясните, как вы пришли к своему ответу.
  2. Частица движется по пунктирной траектории, которая на полностью находится в плоскости страницы (т.е. пунктирная траектория не является спиралью). Это связано с тем, что магнитное поле, создаваемое длинным прямым проводом с током, неоднородно по силе. В четком, последовательном ответе длиной в абзац, который может включать уравнения и / или дополнительные рисунки, объясните, почему путь частицы выглядит так, как указано выше, а не круговым. Обязательно укажите особенности напряженности магнитного поля и того, как эта напряженность поля влияет на движение заряда.

Решение

(a-i) 2 балла

  1. Студент демонстрирует понимание того, что магнитная сила, действующая на частицу, всегда перпендикулярна направлению скорости частицы.
  2. Учащийся демонстрирует понимание того, что чистая сила, действующая перпендикулярно скорости объекта, не может изменить его скорость. Студент может использовать принципы работы и энергии, чтобы привести этот аргумент, но в этом нет необходимости.
Пример:

Магнитное поле может оказывать на заряд только силу, перпендикулярную вектору скорости заряда.Но сила должна быть острой со скоростью, чтобы ускорить объект, или тупой, чтобы замедлить объект. Поскольку сила перпендикулярна скорости, заряд не может изменять скорость.

Пример:

Магнитная сила всегда перпендикулярна скорости. Если сила перпендикулярна движению объекта, эта сила не совершает никакой работы. Следовательно, магнитная сила не может изменять отдачу или принимать кинетическую энергию от объекта, поэтому объект не может ускоряться или замедляться.

Альтернативное решение

  1. Учащийся делает общее заявление, что магнитные поля не могут воздействовать на объект.
  2. Учащийся осознает, что сетевая работа над объектом изменяет кинетическую энергию объекта, поэтому отсутствие чистой работы означает отсутствие изменения кинетической энергии или скорости частицы.
Пример:

Магнитные поля никогда не работают. Поскольку работа вызывает изменение кинетической энергии объекта, отсутствие чистой работы означает отсутствие изменения скорости.

(a-ii) 1 балл

  1. Студент демонстрирует понимание того, что радиус кривизны пути объекта зависит от силы центростремительной силы, и утверждает, что однородное магнитное поле создает постоянную силу на частицу.

(b-i) 2 балла

Частица имеет положительный заряд. Этот вывод должен быть сделан, чтобы заработать оба балла.

  1. Учащийся выбирает место на пунктирной траектории частицы и правильно указывает направление скорости и магнитной силы в этой точке. (Если выбрана «верхняя точка» на пунктирной линии, например P , скорость направлена ​​вправо, а сила – вниз, «к центру» кривизны. Если «нижняя точка» находится ближе всего к Проволока выбрана пунктирной линией, скорость слева, сила вверх.)
  2. Учащийся узнает, что магнитное поле проволоки направлено за пределы страницы в область, где движется частица.

Если высказана одна из этих идей, зарабатывается одно очко. Оба балла зарабатываются только в том случае, если ученик связывает свои направления скорости, магнитного поля и суммарной силы с правильным знаком заряда.

(b-ii) 5 баллов

  1. Учащийся понимает, что радиус кривизны траектории частицы уменьшается по мере приближения частицы к проводу.
  2. Студент объясняет, что магнитное поле сильнее ближе к проводу. Учащийся может использовать уравнение как часть объяснения этой идеи.
  3. Студент объясняет, что чем сильнее магнитное поле, тем сильнее магнитная сила, действующая на частицу. Учащийся может использовать уравнение как часть объяснения этой идеи.
  4. Студент заявляет или подразумевает, что магнитная сила обеспечивает центростремительную силу для частицы.Студент также объясняет, что более сильная центростремительная сила приводит к меньшему радиусу кривизны объекта. Учащийся может использовать уравнение как часть объяснения этой идеи.
  5. Для логического, релевантного и внутренне непротиворечивого ответа, который учитывает заданный аргумент или вопрос require и следует руководящим принципам, описанным в опубликованных требованиях для ответа длиной в абзац.
Пример:

Путь в точке P очень прямой, но вблизи провода путь резко изогнут.Это связано с тем, что магнитное поле рядом с проводом сильнее, оказывая большую силу на заряд. Поскольку магнитное поле всегда перпендикулярно скорости, магнитная сила может только «управлять» направлением заряда. Более сильная сила подобна более сильному повороту рулевого колеса на машине – она ​​делает путь более крутым и имеет меньший радиус. Это то, что происходит вблизи провода, где сильное магнитное поле, но вдали от провода, где поле слабое, заряд не меняет направление так быстро, и путь имеет больший радиус.

Комментарий

Эта проблема возникла в результате бесплатного ответа № 5 на выпущенные бесплатные вопросы для экзамена AP Physics 1 в 2016 году (для краткости P1-2016-FR05). В этой задаче вертикальная струна удерживает стоячую волну, которая имеет разные длины волн в разных местах из-за разного натяжения струны. Мы часто обучаем стоячим волнам в средствах массовой информации, где скорость волны одинакова во всем; это приводит к образцам стоячих волн, имеющих одинаковое расстояние между узлами и пучностями.Точно так же мы часто учим движение заряда через магнитное поле только в частном случае, когда магнитное поле однородно. Эта задача проверяет, понимают ли студенты качественно, как магнитное поле влияет на радиус кривизны пути заряженной частицы, даже если магнитное поле меняется от места к месту.

Задача начинается с утверждения, что магнитная сила не может изменить скорость заряда. Задача состоит в том, чтобы учащиеся могли использовать этот факт позже как часть своего ответа длиной в абзац, даже если они не знают, почему это так (поскольку в части (a) P1-2016-FR05 прямо говорится, что напряжение больше в верхней части веревки и спрашивает, почему, так что студенты, которые не знают почему, все еще могут использовать этот факт позже в части (b)).Вопрос о том, почему однородное поле приводит к круговой траектории, задается для того, чтобы подготовить студентов к сравнению и сопоставлению равномерного кругового движения заряда в однородном B-поле с неравномерным, некруглым движением заряд в неоднородном B-поле, которое возникает на части (b).

Затем вопрос переходит к заряду вблизи длинного прямого провода. В первой части этого задания студентам предлагается указать знак обвинения и объяснить, как они пришли к своему ответу.Многие учителя просто тренируют студентов по таким задачам, как «вот знак заряда, направление скорости и направление магнитного поля, в котором находится заряд; теперь скажите мне направление магнитной силы, действующей на заряд». Хотя это ценная неизбежная часть обучения магнетизму, очень маловероятно, что студенты увидят такой прямой вопрос о применении правила правой руки на экзамене по физике 2. Вместо этого учащимся нужно будет рассмотреть более сложную ситуацию и уметь извлекать из ситуации три из этих вещей (знак заряда, направление скорости заряда, направление магнитного поля и направление магнитной силы) и определять четвертый.

Вторая часть «заряд-и-провод» – это причина, по которой у нас возникла эта проблема: определить, может ли учащийся понять принципы и отношения в контексте изученной темы, но в ситуации, с которой он вряд ли столкнется. перед. В этом случае учащийся должен объединить следующие идеи, чтобы объяснить, почему заряд идет по пунктирной линии:

  • Траектория имеет небольшой радиус кривизны вблизи проволоки и большой радиус кривизны вдали.Во время AP Physics 1 студентов можно научить распознавать длинные и короткие пути с радиусом кривизны. Это можно сделать в контексте дороги и вождения; нарисуйте кривую на доске и скажите, что эта кривая представляет дорогу, если смотреть сверху. Нарисуйте кривую так, чтобы были четкие места, где радиус кривизны большой, а где он мал, и попросите учащихся объяснить, где машины могут ехать быстрее, а где они должны ехать медленнее.
  • Магнитное поле сильнее в местах, расположенных ближе к проводу.Обычно это преподается как часть типичного модуля магнетизма с использованием уравнения для магнитного поля, обусловленного прямым проводом.
  • Более сильное магнитное поле действует на заряд с большей силой. Это преподается как часть обучения уравнению. Обратите внимание, что учащимся необходимо знать, что только B может влиять на значение F, поскольку магнитное поле не может изменить скорость v частицы, а заряд зависит от свойств частицы.
  • Магнитная сила перпендикулярна движению, поэтому магнитная сила обеспечивает центростремительную силу на частицу.
  • Чем сильнее центростремительная сила, действующая на объект, тем более крутой поворот (меньший радиус кривизны) может совершить частица. Это снова следует ввести в Физике 1, опять же в контексте дорог с разным радиусом кривизны в разных местах. Учащиеся должны понимать, что для крутого поворота требуется больше силы, потому что в этих случаях движение меняется быстрее.

Викторина

Четыре частицы движутся вблизи длинного провода, по которому проходит ток I .Частицы имеют массу и заряд, как показано в таблице. Проволока и все частицы находятся в плоскости страницы, как показано на схеме. Стрелка на каждой частице представляет начальное движение этой частицы. Сетка может использоваться для количественной оценки местоположения и относительной скорости частиц. Единственная сила, которая действует на каждую частицу из-за магнитного поля провода.

(a) Какие две частицы испытывают магнитные силы в одном направлении в момент, показанный на диаграмме? Объясните, как вы пришли к своему ответу.

(b) Расположите частицы по величине сил, действующих на эту частицу, от самых сильных к самым слабым. Поместите символ> или = между каждой буквой вашего рейтинга.

Сильная сила ____ ____ _____ _____ Слабая сила

Обоснуйте свой ответ количественно, используя уравнения.

(c) Расположите частицы по величине ускорения каждой частицы от наибольшего к наименьшему.

Обоснуйте свой ответ количественно, используя уравнения.

Максимальное ускорение ____ ____ _____ _____ Наименьшее ускорение

Каждая частица совершает круговое движение в магнитном поле провода. Все круговые движения имеют радиусы, которые очень малы по сравнению с расстоянием частицы от провода , , так что магнитное поле вокруг каждого круга можно считать однородным.

(d) Какая частица или частицы совершают круговое движение против часовой стрелки? Объясните, как вы пришли к своему ответу.

(e) Расположите частицы по радиусу кругового движения каждой частицы от наибольшего к наименьшему.

Обоснуйте свой ответ количественно, используя уравнения.

Наибольший радиус ____ ____ _____ _____ Наименьший радиус

(f) Распределите частицы по периоду кругового движения каждой частицы от самого длинного до самого короткого.

Обоснуйте свой ответ количественно, используя уравнения.

Самый длинный период ____ ____ _____ _____ Самый короткий период

Решения

17 баллов.

(а)

  • Над проводом магнитное поле вне страницы. Магнитное поле находится на странице под проводом.
  • Частица P ощущает силу, направленную влево (положительную, движущуюся вниз, вне магнитного поля страницы).
  • Частица N ощущает восходящую силу (отрицательную, движется влево, в магнитное поле страницы).
  • Частица D ощущает силу, направленную вниз (положительную, движущуюся вправо, вне магнитного поля страницы).
  • Частица A ощущает силу, направленную влево (положительную, движущуюся вверх, в магнитное поле страницы).
  • (1 балл) P и A ощущают одинаковую магнитную силу.
  • (1 балл) Ссылаясь на ручное правило.

(б)

  • (1 балл) Правильный рейтинг: N = D > A > P
  • (1 балл) Указывает, что магнитное поле пропорционально 1/ y , где y – это расстояние заряда от провода.Использование уравнения удовлетворило бы это требование.
  • (1 балл) Это означает.
  • (1 балл) Сложив эти уравнения вместе, мы обнаружим, что сила пропорциональна. Формулировка полного уравнения удовлетворяет последним трем пунктам.

(в)

  • (1 балл) Правильный рейтинг: N > D > P > A
  • (1 балл) Изложение и использование второго закона Ньютона,. Использование НЕ приносит этот балл.
  • (1 балл) Используя предыдущее выражение для силы, найти ускорение, пропорциональное. Формулировка полного уравнения удовлетворяет последним трем пунктам.

(г)

  • (1 балл) – единственная частица, которая движется против часовой стрелки.
  • (1 балл) Учащийся указывает, что направление силы должно быть на 90 o против часовой стрелки от направления скорости для кругового движения против часовой стрелки, или какой-либо другой убедительный аргумент.

e)

  • (1 балл) Правильный рейтинг: D = A > P = N
  • (1 балл) Установка магнитной силы равной центростремительной силе, или установка ускорения, указанного в (c), равным.
  • (1 балл) Решение для радиуса и определение того, что радиус пропорционален или дает полное уравнение для r , которое показывает mvy в числителе и q в знаменателе.

(ж)

  • (1 балл) Правильный рейтинг: A > P = D > N
  • (1 балл) Используя уравнение.
  • (1 балл) Решение для периода к этому периоду пропорционально или какому-либо другому уравнению, которое показывает эту взаимосвязь.

Автор

Джон Френсли
Средняя школа Проспера
Проспер, Техас

11.4: Движение заряженной частицы в магнитном поле

Цели обучения

К концу этого раздела вы сможете:

  • Объясните, как заряженная частица во внешнем магнитном поле совершает круговое движение
  • Опишите, как определить радиус кругового движения заряженной частицы в магнитном поле

Заряженная частица испытывает силу при движении в магнитном поле.Что произойдет, если это поле будет однородным при движении заряженной частицы? По какому пути следует частица? В этом разделе мы обсуждаем круговое движение заряженной частицы, а также другое движение, возникающее в результате попадания заряженной частицы в магнитное поле.

Самый простой случай имеет место, когда заряженная частица движется перпендикулярно однородному полю B (рисунок \ (\ PageIndex {1} \)). Если поле находится в вакууме, магнитное поле является доминирующим фактором, определяющим движение.Поскольку магнитная сила перпендикулярна направлению движения, заряженная частица следует по кривой траектории в магнитном поле. Частица продолжает следовать по этому изогнутому пути, пока не образует полный круг. Другой способ взглянуть на это состоит в том, что магнитная сила всегда перпендикулярна скорости, поэтому она не действует на заряженную частицу. Таким образом, кинетическая энергия и скорость частицы остаются постоянными. Это влияет на направление движения, но не на скорость.

Рисунок \ (\ PageIndex {1} \): отрицательно заряженная частица движется в плоскости бумаги в области, где магнитное поле перпендикулярно бумаге (представленное маленькими \ (X \) – как хвосты стрелок).2} {r}. \]

Решение для r дает

\ [r = \ dfrac {mv} {qB}. \ label {11.5} \]

Здесь r – радиус кривизны пути заряженной частицы с массой м и зарядом q , движущейся со скоростью v , перпендикулярной магнитному полю с напряженностью B . Время прохождения заряженной частицы по круговой траектории определяется как период, равный пройденному расстоянию (окружности), деленному на скорость.Основываясь на этом и уравнении, мы можем получить период движения как

.

\ [T = \ dfrac {2 \ pi r} {v} = \ dfrac {2 \ pi} {v} \ dfrac {mv} {qB} = \ dfrac {2 \ pi m} {qB}. \ label {11.6} \]

Если скорость не перпендикулярна магнитному полю, то мы можем сравнить каждую составляющую скорости отдельно с магнитным полем. Компонент скорости, перпендикулярный магнитному полю, создает магнитную силу, перпендикулярную как этой скорости, так и полю:

\ [\ begin {align} v_ {perp} & = v \, \ sin \ theta \\ [4pt] v_ {para} & = v \, \ cos \ theta.\ end {align} \]

, где \ (\ theta \) – угол между v и B . Компонент, параллельный магнитному полю, создает постоянное движение в том же направлении, что и магнитное поле, что также показано в уравнении. Параллельное движение определяет шаг p спирали, то есть расстояние между соседними витками. Это расстояние равно параллельной составляющей скорости, умноженной на период:

\ [p = v_ {para} T. \ label {11.8} \]

Результатом будет винтового движения , как показано на следующем рисунке.

Рисунок \ (\ PageIndex {2} \): Заряженная частица движется со скоростью, отличной от направления магнитного поля. Компонент скорости, перпендикулярный магнитному полю, создает круговое движение, тогда как компонент скорости, параллельный полю, перемещает частицу по прямой. Шаг – это расстояние по горизонтали между двумя последовательными кругами. Результирующее движение – спиральное.

Пока заряженная частица движется по спирали, она может попасть в область, где магнитное поле неоднородно.В частности, предположим, что частица перемещается из области сильного магнитного поля в область более слабого поля, а затем обратно в область более сильного поля. Частица может отразиться до того, как войдет в область с более сильным магнитным полем. Это похоже на волну на струне, которая движется от очень легкой тонкой струны к твердой стене и отражается назад. Если отражение происходит с обоих концов, частица оказывается в так называемой магнитной бутылке.

Захваченные частицы в магнитных полях обнаружены в радиационных поясах Ван Аллена вокруг Земли, которые являются частью магнитного поля Земли.Эти пояса были обнаружены Джеймсом Ван Алленом при попытке измерить поток космических лучей на Земле (частицы высокой энергии, приходящие извне Солнечной системы), чтобы выяснить, похож ли он на поток, измеренный на Земле. Ван Аллен обнаружил, что из-за вклада частиц, захваченных магнитным полем Земли, поток на Земле был намного выше, чем в космическом пространстве. Полярные сияния, как и знаменитое полярное сияние (северное сияние) в Северном полушарии (рис. \ (\ PageIndex {3} \)), представляют собой прекрасные проявления света, излучаемого при рекомбинации ионов с электронами, входящими в атмосферу, когда они вращаются вдоль силовых линий магнитного поля.(Ионы – это в основном атомы кислорода и азота, которые первоначально ионизируются в результате столкновений с энергичными частицами в атмосфере Земли.) Полярные сияния также наблюдались на других планетах, таких как Юпитер и Сатурн.

Рисунок \ (\ PageIndex {3} \): (a) Радиационные пояса Ван Аллена вокруг Земли захватывают ионы, образованные космическими лучами, падающими на атмосферу Земли. (b) Великолепное зрелище северного сияния, или северного сияния, сияет в северном небе над Беар-Лейк недалеко от базы ВВС Эйлсон, Аляска.Этот свет, сформированный магнитным полем Земли, создается светящимися молекулами и ионами кислорода и азота. (кредит b: модификация работы старшего летчика ВВС США Джошуа Стрэнга)

Пример \ (\ PageIndex {1} \): отражатель луча

Группа исследователей занимается изучением короткоживущих радиоактивных изотопов. Им необходимо разработать способ транспортировки альфа-частиц (ядер гелия) от места их создания к месту, где они столкнутся с другим материалом, образуя изотоп. Пучок альфа-частиц \ ((m = 6.{-19} C) \) изгибается через область под углом 90 градусов с однородным магнитным полем 0,050 Тл (рисунок \ (\ PageIndex {4} \)). а) В каком направлении следует приложить магнитное поле? (б) Сколько времени требуется альфа-частицам, чтобы пройти через область однородного магнитного поля?

Рисунок \ (\ PageIndex {4} \): вид сверху на установку дефлектора балки.

Стратегия

  1. Направление магнитного поля показано RHR-1. Ваши пальцы указывают в направлении v , а большой палец должен указывать в направлении силы, влево.Следовательно, поскольку альфа-частицы заряжены положительно, магнитное поле должно указывать вниз. {- 27} кг)} {(3.{-7} с. \]

Значение

Это время может быть достаточно быстрым, чтобы добраться до материала, который мы хотели бы бомбардировать, в зависимости от того, насколько короткоживущий радиоактивный изотоп и продолжает испускать альфа-частицы. Если бы мы могли усилить магнитное поле, приложенное к области, это сократило бы время еще больше. Путь, по которому частицы должны пройти, можно было бы сократить, но это может оказаться неэкономичным, учитывая экспериментальную установку.

Упражнение \ (\ PageIndex {1} \)

Однородное магнитное поле величиной 1.5 \, м / с \). Под каким углом должно быть магнитное поле относительно скорости, чтобы шаг результирующего спирального движения был равен радиусу спирали?

Стратегия

Шаг движения относится к параллельной скорости, умноженной на период кругового движения, тогда как радиус относится к перпендикулярной составляющей скорости. После установки равных друг другу радиуса и шага найдите угол между магнитным полем и скоростью или \ (\ theta \).

Решение

Шаг задается уравнением \ ref {11.8}, период задается уравнением \ ref {11.6}, а радиус кругового движения задается уравнением \ ref {11.5}. Обратите внимание, что скорость в уравнении радиуса связана только с перпендикулярной скоростью, в которой происходит круговое движение. Поэтому мы подставляем синусоидальную составляющую общей скорости в уравнение радиуса, чтобы приравнять шаг и радиус

\ [p = r \]

\ [v _ {\ parallel} T = \ dfrac {mv} {qB} \]

\ [v \, cos \, \ theta \ dfrac {2 \ pi m} {qB} = \ dfrac {mv \, sin \, \ theta} {qB} \]

\ [2 \ pi = загар \, \ theta \]

\ [\ theta = 81.о \) будет происходить только круговое движение, и не будет движения кругов перпендикулярно движению. Вот что создает спиральное движение.

Авторы и авторство

  • Сэмюэл Дж. Линг (Государственный университет Трумэна), Джефф Санни (Университет Лойола Мэримаунт) и Билл Мобс со многими авторами.

Оставить комментарий